Evaluate...












0












$begingroup$


I'd like to Prove that $sumlimits_{n=0}^{infty}sumlimits_{r=0}^{n}left(frac{1}{(n-r)!}a^{n-r}right)left(frac{1}{r!}b^{r}right)=left(sumlimits_{n=0}^{infty}frac{1}{n!}a^nright)left(sumlimits_{n=0}^{infty}frac{1}{n!}b^nright)$



I do as follow




$sumlimits_{n=0}^{infty}sumlimits_{r=0}^{n}left(frac{1}{(n-r)!}a^{n-r}right)left(frac{1}{r!}b^{r}right)=sumlimits_{r=0}^{0}left(frac{1}{(0-r)!}a^{0-r}right)left(frac{1}{r!}b^{r}right)+sumlimits_{r=0}^{1}left(frac{1}{(1-r)!}a^{1-r}right)left(frac{1}{r!}b^{r}right)+sumlimits_{r=0}^{2}left(frac{1}{(2-r)!}a^{2-r}right)left(frac{1}{r!}b^{r}right)+cdots$




I couldn't able to get the right hand



Any help will be appreciated! Thanks










share|cite|improve this question









$endgroup$

















    0












    $begingroup$


    I'd like to Prove that $sumlimits_{n=0}^{infty}sumlimits_{r=0}^{n}left(frac{1}{(n-r)!}a^{n-r}right)left(frac{1}{r!}b^{r}right)=left(sumlimits_{n=0}^{infty}frac{1}{n!}a^nright)left(sumlimits_{n=0}^{infty}frac{1}{n!}b^nright)$



    I do as follow




    $sumlimits_{n=0}^{infty}sumlimits_{r=0}^{n}left(frac{1}{(n-r)!}a^{n-r}right)left(frac{1}{r!}b^{r}right)=sumlimits_{r=0}^{0}left(frac{1}{(0-r)!}a^{0-r}right)left(frac{1}{r!}b^{r}right)+sumlimits_{r=0}^{1}left(frac{1}{(1-r)!}a^{1-r}right)left(frac{1}{r!}b^{r}right)+sumlimits_{r=0}^{2}left(frac{1}{(2-r)!}a^{2-r}right)left(frac{1}{r!}b^{r}right)+cdots$




    I couldn't able to get the right hand



    Any help will be appreciated! Thanks










    share|cite|improve this question









    $endgroup$















      0












      0








      0


      1



      $begingroup$


      I'd like to Prove that $sumlimits_{n=0}^{infty}sumlimits_{r=0}^{n}left(frac{1}{(n-r)!}a^{n-r}right)left(frac{1}{r!}b^{r}right)=left(sumlimits_{n=0}^{infty}frac{1}{n!}a^nright)left(sumlimits_{n=0}^{infty}frac{1}{n!}b^nright)$



      I do as follow




      $sumlimits_{n=0}^{infty}sumlimits_{r=0}^{n}left(frac{1}{(n-r)!}a^{n-r}right)left(frac{1}{r!}b^{r}right)=sumlimits_{r=0}^{0}left(frac{1}{(0-r)!}a^{0-r}right)left(frac{1}{r!}b^{r}right)+sumlimits_{r=0}^{1}left(frac{1}{(1-r)!}a^{1-r}right)left(frac{1}{r!}b^{r}right)+sumlimits_{r=0}^{2}left(frac{1}{(2-r)!}a^{2-r}right)left(frac{1}{r!}b^{r}right)+cdots$




      I couldn't able to get the right hand



      Any help will be appreciated! Thanks










      share|cite|improve this question









      $endgroup$




      I'd like to Prove that $sumlimits_{n=0}^{infty}sumlimits_{r=0}^{n}left(frac{1}{(n-r)!}a^{n-r}right)left(frac{1}{r!}b^{r}right)=left(sumlimits_{n=0}^{infty}frac{1}{n!}a^nright)left(sumlimits_{n=0}^{infty}frac{1}{n!}b^nright)$



      I do as follow




      $sumlimits_{n=0}^{infty}sumlimits_{r=0}^{n}left(frac{1}{(n-r)!}a^{n-r}right)left(frac{1}{r!}b^{r}right)=sumlimits_{r=0}^{0}left(frac{1}{(0-r)!}a^{0-r}right)left(frac{1}{r!}b^{r}right)+sumlimits_{r=0}^{1}left(frac{1}{(1-r)!}a^{1-r}right)left(frac{1}{r!}b^{r}right)+sumlimits_{r=0}^{2}left(frac{1}{(2-r)!}a^{2-r}right)left(frac{1}{r!}b^{r}right)+cdots$




      I couldn't able to get the right hand



      Any help will be appreciated! Thanks







      calculus summation






      share|cite|improve this question













      share|cite|improve this question











      share|cite|improve this question




      share|cite|improve this question










      asked Dec 17 '18 at 6:53









      user62498user62498

      1,983714




      1,983714






















          3 Answers
          3






          active

          oldest

          votes


















          0












          $begingroup$

          You might find it easier to start from the RHS and show that



          $$ left(sum_{n=0}^{infty}frac{1}{n!}a^nright)left(sum_{n=0}^{infty}frac{1}{n!}b^nright) = sumlimits_{n=0}^{infty}sum_{r=0}^{n}left(frac{1}{(n-r)!}a^{n-r}right)left(frac{1}{r!}b^{r}right). $$



          Actually, for me this is the only step. It's just how you multiply two series.



          Something more interesting would be to see what you can make of



          $$ frac{1}{(n-r)!r!}a^{n - r}b^r = frac{1}{n!} binom{n}{r} a^{n - r}b^r $$



          and the Binomial Theorem.






          share|cite|improve this answer









          $endgroup$





















            1












            $begingroup$

            Hint



            Consider the inner summation and show that $$sumlimits_{r=0}^{n}left(frac{a^{n-r}}{(n-r)!}right)left(frac{b^{r}}{r!}right)=frac{(a+b)^n}{n!}$$ Now, the lhs will be very familiar to you and the remaining is simple.






            share|cite|improve this answer









            $endgroup$













            • $begingroup$
              I do it how can I get $sum_{n=0}^{infty}frac{(a+b)^n}{n!}=$ right hand
              $endgroup$
              – user62498
              Dec 17 '18 at 7:37





















            0












            $begingroup$

            By hint Claude Leibovici,




            $ sumlimits_{n=0}^{infty}sumlimits_{r=0}^{n}left(frac{1}{(n-r)!}a^{n-r}right)left(frac{1}{r!}b^{r}right)=sumlimits_{n=0}^{infty}frac{1}{n!}(a+b)^n=e^{a+b}$ on the other hand



            $e^{a+b}=e^{a}e^{b}=left(sumlimits_{n=0}^{infty}frac{1}{n!}(a)^nright)left(sumlimits_{n=0}^{infty}frac{1}{n!}(b)^nright)$




            Please let me know if this makes sense to you






            share|cite|improve this answer









            $endgroup$













              Your Answer





              StackExchange.ifUsing("editor", function () {
              return StackExchange.using("mathjaxEditing", function () {
              StackExchange.MarkdownEditor.creationCallbacks.add(function (editor, postfix) {
              StackExchange.mathjaxEditing.prepareWmdForMathJax(editor, postfix, [["$", "$"], ["\\(","\\)"]]);
              });
              });
              }, "mathjax-editing");

              StackExchange.ready(function() {
              var channelOptions = {
              tags: "".split(" "),
              id: "69"
              };
              initTagRenderer("".split(" "), "".split(" "), channelOptions);

              StackExchange.using("externalEditor", function() {
              // Have to fire editor after snippets, if snippets enabled
              if (StackExchange.settings.snippets.snippetsEnabled) {
              StackExchange.using("snippets", function() {
              createEditor();
              });
              }
              else {
              createEditor();
              }
              });

              function createEditor() {
              StackExchange.prepareEditor({
              heartbeatType: 'answer',
              autoActivateHeartbeat: false,
              convertImagesToLinks: true,
              noModals: true,
              showLowRepImageUploadWarning: true,
              reputationToPostImages: 10,
              bindNavPrevention: true,
              postfix: "",
              imageUploader: {
              brandingHtml: "Powered by u003ca class="icon-imgur-white" href="https://imgur.com/"u003eu003c/au003e",
              contentPolicyHtml: "User contributions licensed under u003ca href="https://creativecommons.org/licenses/by-sa/3.0/"u003ecc by-sa 3.0 with attribution requiredu003c/au003e u003ca href="https://stackoverflow.com/legal/content-policy"u003e(content policy)u003c/au003e",
              allowUrls: true
              },
              noCode: true, onDemand: true,
              discardSelector: ".discard-answer"
              ,immediatelyShowMarkdownHelp:true
              });


              }
              });














              draft saved

              draft discarded


















              StackExchange.ready(
              function () {
              StackExchange.openid.initPostLogin('.new-post-login', 'https%3a%2f%2fmath.stackexchange.com%2fquestions%2f3043618%2fevaluate-sum-limits-n-0-infty-sum-limits-r-0n-left-frac1n-ra%23new-answer', 'question_page');
              }
              );

              Post as a guest















              Required, but never shown

























              3 Answers
              3






              active

              oldest

              votes








              3 Answers
              3






              active

              oldest

              votes









              active

              oldest

              votes






              active

              oldest

              votes









              0












              $begingroup$

              You might find it easier to start from the RHS and show that



              $$ left(sum_{n=0}^{infty}frac{1}{n!}a^nright)left(sum_{n=0}^{infty}frac{1}{n!}b^nright) = sumlimits_{n=0}^{infty}sum_{r=0}^{n}left(frac{1}{(n-r)!}a^{n-r}right)left(frac{1}{r!}b^{r}right). $$



              Actually, for me this is the only step. It's just how you multiply two series.



              Something more interesting would be to see what you can make of



              $$ frac{1}{(n-r)!r!}a^{n - r}b^r = frac{1}{n!} binom{n}{r} a^{n - r}b^r $$



              and the Binomial Theorem.






              share|cite|improve this answer









              $endgroup$


















                0












                $begingroup$

                You might find it easier to start from the RHS and show that



                $$ left(sum_{n=0}^{infty}frac{1}{n!}a^nright)left(sum_{n=0}^{infty}frac{1}{n!}b^nright) = sumlimits_{n=0}^{infty}sum_{r=0}^{n}left(frac{1}{(n-r)!}a^{n-r}right)left(frac{1}{r!}b^{r}right). $$



                Actually, for me this is the only step. It's just how you multiply two series.



                Something more interesting would be to see what you can make of



                $$ frac{1}{(n-r)!r!}a^{n - r}b^r = frac{1}{n!} binom{n}{r} a^{n - r}b^r $$



                and the Binomial Theorem.






                share|cite|improve this answer









                $endgroup$
















                  0












                  0








                  0





                  $begingroup$

                  You might find it easier to start from the RHS and show that



                  $$ left(sum_{n=0}^{infty}frac{1}{n!}a^nright)left(sum_{n=0}^{infty}frac{1}{n!}b^nright) = sumlimits_{n=0}^{infty}sum_{r=0}^{n}left(frac{1}{(n-r)!}a^{n-r}right)left(frac{1}{r!}b^{r}right). $$



                  Actually, for me this is the only step. It's just how you multiply two series.



                  Something more interesting would be to see what you can make of



                  $$ frac{1}{(n-r)!r!}a^{n - r}b^r = frac{1}{n!} binom{n}{r} a^{n - r}b^r $$



                  and the Binomial Theorem.






                  share|cite|improve this answer









                  $endgroup$



                  You might find it easier to start from the RHS and show that



                  $$ left(sum_{n=0}^{infty}frac{1}{n!}a^nright)left(sum_{n=0}^{infty}frac{1}{n!}b^nright) = sumlimits_{n=0}^{infty}sum_{r=0}^{n}left(frac{1}{(n-r)!}a^{n-r}right)left(frac{1}{r!}b^{r}right). $$



                  Actually, for me this is the only step. It's just how you multiply two series.



                  Something more interesting would be to see what you can make of



                  $$ frac{1}{(n-r)!r!}a^{n - r}b^r = frac{1}{n!} binom{n}{r} a^{n - r}b^r $$



                  and the Binomial Theorem.







                  share|cite|improve this answer












                  share|cite|improve this answer



                  share|cite|improve this answer










                  answered Dec 17 '18 at 7:12









                  Trevor GunnTrevor Gunn

                  14.9k32047




                  14.9k32047























                      1












                      $begingroup$

                      Hint



                      Consider the inner summation and show that $$sumlimits_{r=0}^{n}left(frac{a^{n-r}}{(n-r)!}right)left(frac{b^{r}}{r!}right)=frac{(a+b)^n}{n!}$$ Now, the lhs will be very familiar to you and the remaining is simple.






                      share|cite|improve this answer









                      $endgroup$













                      • $begingroup$
                        I do it how can I get $sum_{n=0}^{infty}frac{(a+b)^n}{n!}=$ right hand
                        $endgroup$
                        – user62498
                        Dec 17 '18 at 7:37


















                      1












                      $begingroup$

                      Hint



                      Consider the inner summation and show that $$sumlimits_{r=0}^{n}left(frac{a^{n-r}}{(n-r)!}right)left(frac{b^{r}}{r!}right)=frac{(a+b)^n}{n!}$$ Now, the lhs will be very familiar to you and the remaining is simple.






                      share|cite|improve this answer









                      $endgroup$













                      • $begingroup$
                        I do it how can I get $sum_{n=0}^{infty}frac{(a+b)^n}{n!}=$ right hand
                        $endgroup$
                        – user62498
                        Dec 17 '18 at 7:37
















                      1












                      1








                      1





                      $begingroup$

                      Hint



                      Consider the inner summation and show that $$sumlimits_{r=0}^{n}left(frac{a^{n-r}}{(n-r)!}right)left(frac{b^{r}}{r!}right)=frac{(a+b)^n}{n!}$$ Now, the lhs will be very familiar to you and the remaining is simple.






                      share|cite|improve this answer









                      $endgroup$



                      Hint



                      Consider the inner summation and show that $$sumlimits_{r=0}^{n}left(frac{a^{n-r}}{(n-r)!}right)left(frac{b^{r}}{r!}right)=frac{(a+b)^n}{n!}$$ Now, the lhs will be very familiar to you and the remaining is simple.







                      share|cite|improve this answer












                      share|cite|improve this answer



                      share|cite|improve this answer










                      answered Dec 17 '18 at 7:15









                      Claude LeiboviciClaude Leibovici

                      124k1157135




                      124k1157135












                      • $begingroup$
                        I do it how can I get $sum_{n=0}^{infty}frac{(a+b)^n}{n!}=$ right hand
                        $endgroup$
                        – user62498
                        Dec 17 '18 at 7:37




















                      • $begingroup$
                        I do it how can I get $sum_{n=0}^{infty}frac{(a+b)^n}{n!}=$ right hand
                        $endgroup$
                        – user62498
                        Dec 17 '18 at 7:37


















                      $begingroup$
                      I do it how can I get $sum_{n=0}^{infty}frac{(a+b)^n}{n!}=$ right hand
                      $endgroup$
                      – user62498
                      Dec 17 '18 at 7:37






                      $begingroup$
                      I do it how can I get $sum_{n=0}^{infty}frac{(a+b)^n}{n!}=$ right hand
                      $endgroup$
                      – user62498
                      Dec 17 '18 at 7:37













                      0












                      $begingroup$

                      By hint Claude Leibovici,




                      $ sumlimits_{n=0}^{infty}sumlimits_{r=0}^{n}left(frac{1}{(n-r)!}a^{n-r}right)left(frac{1}{r!}b^{r}right)=sumlimits_{n=0}^{infty}frac{1}{n!}(a+b)^n=e^{a+b}$ on the other hand



                      $e^{a+b}=e^{a}e^{b}=left(sumlimits_{n=0}^{infty}frac{1}{n!}(a)^nright)left(sumlimits_{n=0}^{infty}frac{1}{n!}(b)^nright)$




                      Please let me know if this makes sense to you






                      share|cite|improve this answer









                      $endgroup$


















                        0












                        $begingroup$

                        By hint Claude Leibovici,




                        $ sumlimits_{n=0}^{infty}sumlimits_{r=0}^{n}left(frac{1}{(n-r)!}a^{n-r}right)left(frac{1}{r!}b^{r}right)=sumlimits_{n=0}^{infty}frac{1}{n!}(a+b)^n=e^{a+b}$ on the other hand



                        $e^{a+b}=e^{a}e^{b}=left(sumlimits_{n=0}^{infty}frac{1}{n!}(a)^nright)left(sumlimits_{n=0}^{infty}frac{1}{n!}(b)^nright)$




                        Please let me know if this makes sense to you






                        share|cite|improve this answer









                        $endgroup$
















                          0












                          0








                          0





                          $begingroup$

                          By hint Claude Leibovici,




                          $ sumlimits_{n=0}^{infty}sumlimits_{r=0}^{n}left(frac{1}{(n-r)!}a^{n-r}right)left(frac{1}{r!}b^{r}right)=sumlimits_{n=0}^{infty}frac{1}{n!}(a+b)^n=e^{a+b}$ on the other hand



                          $e^{a+b}=e^{a}e^{b}=left(sumlimits_{n=0}^{infty}frac{1}{n!}(a)^nright)left(sumlimits_{n=0}^{infty}frac{1}{n!}(b)^nright)$




                          Please let me know if this makes sense to you






                          share|cite|improve this answer









                          $endgroup$



                          By hint Claude Leibovici,




                          $ sumlimits_{n=0}^{infty}sumlimits_{r=0}^{n}left(frac{1}{(n-r)!}a^{n-r}right)left(frac{1}{r!}b^{r}right)=sumlimits_{n=0}^{infty}frac{1}{n!}(a+b)^n=e^{a+b}$ on the other hand



                          $e^{a+b}=e^{a}e^{b}=left(sumlimits_{n=0}^{infty}frac{1}{n!}(a)^nright)left(sumlimits_{n=0}^{infty}frac{1}{n!}(b)^nright)$




                          Please let me know if this makes sense to you







                          share|cite|improve this answer












                          share|cite|improve this answer



                          share|cite|improve this answer










                          answered Dec 17 '18 at 8:04









                          user62498user62498

                          1,983714




                          1,983714






























                              draft saved

                              draft discarded




















































                              Thanks for contributing an answer to Mathematics Stack Exchange!


                              • Please be sure to answer the question. Provide details and share your research!

                              But avoid



                              • Asking for help, clarification, or responding to other answers.

                              • Making statements based on opinion; back them up with references or personal experience.


                              Use MathJax to format equations. MathJax reference.


                              To learn more, see our tips on writing great answers.




                              draft saved


                              draft discarded














                              StackExchange.ready(
                              function () {
                              StackExchange.openid.initPostLogin('.new-post-login', 'https%3a%2f%2fmath.stackexchange.com%2fquestions%2f3043618%2fevaluate-sum-limits-n-0-infty-sum-limits-r-0n-left-frac1n-ra%23new-answer', 'question_page');
                              }
                              );

                              Post as a guest















                              Required, but never shown





















































                              Required, but never shown














                              Required, but never shown












                              Required, but never shown







                              Required, but never shown

































                              Required, but never shown














                              Required, but never shown












                              Required, but never shown







                              Required, but never shown







                              Popular posts from this blog

                              Plaza Victoria

                              In PowerPoint, is there a keyboard shortcut for bulleted / numbered list?

                              How to put 3 figures in Latex with 2 figures side by side and 1 below these side by side images but in...